LSAT and Law School Admissions Forum

Get expert LSAT preparation and law school admissions advice from PowerScore Test Preparation.

User avatar
 Dave Killoran
PowerScore Staff
  • PowerScore Staff
  • Posts: 5853
  • Joined: Mar 25, 2011
|
#46103
Complete Question Explanation
(The complete setup for this game can be found here: lsat/viewtopic.php?t=8712)

The correct answer choice is (C)

The first step is to apply the VY block rule because, if one does not see Hitchcock, it is certain the other will not see Hitchcock either. Answer choice (D) contains Y, but not V, and is incorrect.

The second step is to consider the Numerical Distributions. In the 1-2-4 distribution, two film buffs see the Hitchcock film, and five film buffs do not see the Hitchcock film. In the 2-4-1 distribution, four film buffs see the Hitchcock film, and three film buffs do not see the Hitchcock film. Thus, since this question stem asks for a complete and accurate list of the film buffs who do NOT see the Hitchcock film, the correct answer choice must contain either three or five film buffs. Since answer choices (A) and (B) contain only two film buffs, they can both be rejected without further analysis.

Since answer choices (C), (D), and (E) each contain three film buffs, it is apparent that they are each generated by the 2-4-1 distribution. In the 2-4-1 distribution we have already ascertained that L, V, and Y each see the Hitchcock film, and so any answer choice that contains L, V, or Y will be incorrect. This eliminates answer choice (D) and again eliminates answer choice (E). Answer choice (C) is therefore correct.
 dwoodsbud08
  • Posts: 2
  • Joined: Sep 08, 2011
|
#1769
I have a question: In the Powerscore Logic Games Bible (the one published last year) on page 343, Game 2 (about the seven film buffs) in the Advanced Features and Techniques question is confusing me. In question in the game I am having problems with is question 15.

The correct answer in the book was C: Ginnie, Ian and Reveka, but I don't understand why. From my understanding, Ian didn't have to be one of the film buffs who did NOT see the Hitchcock film. He could have seen an Hitchcock film and Marcos could have been the person who did not see it. As a result, I marked the answer B.

The book explains it as there must be three people who didn't see the film because only 4 people could. However, either Ian or Marcos could have seen the Hitchcock film.

Could you please explain to me in further detail why the answer is C.
User avatar
 Dave Killoran
PowerScore Staff
  • PowerScore Staff
  • Posts: 5853
  • Joined: Mar 25, 2011
|
#1770
Hey Dwood,

I think the question stem is what is getting you in trouble here. The question wants you to supply a list of people in a single solution who do not see H. Thus, you aren't trying to list everyone who at anytime doesn't have to see H.

As the text in the book explains, because either 2 or 4 people must see H, either 3 or 5 people always do not see H. Thus, for an answer to be correct, it must contain 3 or 5 people. (A) and (B) contain only 2 people, and so they cannot be a complete list of the people who do not see H in a solution.

In (C), those three can be the three people not seeing H in a solution, and so that is the correct answer.

Does that make sense? Please let me know.

Thanks!
 dwoodsbud08
  • Posts: 2
  • Joined: Sep 08, 2011
|
#1778
Dave,

I understand now. It is simply asking for a hypothetical, and not only the definite ones.

Thanks
 Mustafaabdulmalek
  • Posts: 19
  • Joined: Nov 17, 2015
|
#21810
why I can not see Hitchcock film ?
F(g,m) H(l,v,y,i) K(R)
 Jon Denning
PowerScore Staff
  • PowerScore Staff
  • Posts: 904
  • Joined: Apr 11, 2011
|
#21834
Hey Mustafa,

Thanks for the question! I feel like the difficulty here might have arisen from a misreading of the question. We're simply asked which answer COULD be a list of people not on H. That doesn't mean I can never go on H, only that here it's possible that G, I, and R all go somewhere else.

First, you'll note that there are two paths this game can take based on the first rule, where H must have twice as many people as F. That creates templates with either a 1-2-4 or a 2-4-1 distribution for F, H, K respectively. So when we start looking at answers for #15 about who is NOT on H, we must have either 5 people not on H (H gets 2) or 3 people not on H (H gets 4). Since there are no 5-person answers, then we must be dealing with the 2-4-1 distribution. That also means (A) and (B) are immediately out, since both would leave H with 5 people and that can never happen.

So how could we make it the case that H does not get G, I, or R...in other words H gets the other four, L, M, V, and Y? Let's try it:

..... Y
..... V
I ..... M
R ..... L ..... G
F ..... H ..... K

And there you go (sorry for some weird spacing)! In that possible configuration H does not get G, I, or R, and we're still good! So that's why answer choice (C) is correct.

I hope that helps!

Jon
 ava17
  • Posts: 18
  • Joined: Jan 12, 2019
|
#61926
How do we know whether the question is asking for a particular scenario in which these variables see J, i.e. if these variables see H at the same time OR whether the question is asking for a general list of variables who don't see H? Thanks!
User avatar
 Dave Killoran
PowerScore Staff
  • PowerScore Staff
  • Posts: 5853
  • Joined: Mar 25, 2011
|
#61935
ava17 wrote:How do we know whether the question is asking for a particular scenario in which these variables see J, i.e. if these variables see H at the same time OR whether the question is asking for a general list of variables who don't see H? Thanks!
Hi Ava,

It's the use of "could be a" that tells you it's a list that fits one scenario. If they meant for the full list of all variables who never see the Hitchcock film, it would likely have been worded in more absolute terms such as, "Which one of the following is the complete and accurate list of the film buffs who cannot see the Hitchcock film?"

As small difference, but obviously enough for LSAC to use as the basis for the correct answer!

Get the most out of your LSAT Prep Plus subscription.

Analyze and track your performance with our Testing and Analytics Package.